Index notation/ Tensors, basic algebra questions.

Click For Summary
The discussion focuses on deriving S in terms of T using index notation and tensors in R^3. The user grapples with the consistency of dummy indices when manipulating equations, specifically questioning whether choosing T_{jj} instead of T_{ii} affects the outcome. Clarifications reveal that dummy indices can be renamed without consequence, although maintaining clarity for readers is advised. The factor of three arises from summing over the Kronecker delta, leading to the conclusion that T_{ii} equals (μ + 3λ)S_{ii}. Ultimately, the user realizes that renaming indices does not necessitate changing corresponding terms in the equations.
binbagsss
Messages
1,291
Reaction score
12
Ok I have T_{ij}=μS_{ij} + λ δ_{ij}δS_{kk}.
I am working in R^3.

(I am after S in terms of T) . I multiply by δ_{ij} to attain:

δ_{ij}T_{ij}=δ_{ij}μS_{ij} + δ_{ij} λ δ_{ij}δT_{kk}

=> T_{jj}=δ_{jj}λS_{kk}+μS_{jj} *

My question is , for the LH term of * I choose T_{jj} rathen than T_{ii}. I then get the same decision for μS_{jj} or μS _{ii} on the last term on the RHS. Does this decision need to be consistent with each other?

Next/Main question...
The solution then continues to attain
T_{kk}=(μ+3λ)S_{jj}

Which I can not see how we have got to. δ_{jj}=3, so for RHS of * I get : 3λS_{kk}+μS_{jj} .

I then rename j and k, to get T_{kk} = 3λS_{jj}+μS_{kk}.
My 'S's' do not have the same dummy indice?

Many Thanks to anyone who can shed some light on this !
 
Physics news on Phys.org
looks like you're having problems with the implicit sums. when in
doubt write it out.

T_{ij}=\mu S_{ij} + \lambda \delta_{ij} \sum_k \delta S_{kk}

You then hit by a \delta_{ij} and sum over both i and j

\sum_{ij} \delta_{ij} T_{ij}=\mu \sum_{ij} \delta_{ij} S_{ij} + \lambda \sum_{ij} \delta_{ij} \delta_{ij} \sum_k \delta S_{kk},

Using the property that \delta_{ij} = 0 unless i = j, and is 1 if i=j, we have

\sum_{i} T_{ii}=\mu \sum_{i} S_{ii} + \lambda \sum_{i} \delta_{ii} \sum_k \delta S_{kk},

Answer 1. it doesn't matter what your dummy index is i, j, k. it's a dummy index. however if you
want to be kind to somebody reading you should make the indices match on both sides of an equation.

Answer 2. the factor of three comes from
\sum_{i} \delta_{ii} = \delta_{11} + \delta_{22} + \delta_{33}= 1+1+1 = 3

This leaves
\sum_{i} T_{ii}=\mu \sum_{i} S_{ii} + 3 \lambda \sum_k \delta S_{kk},

Does it matter that the summation indices are different? not really but it is ugly. So use
a different letter!
\sum_{i} T_{ii}=\mu \sum_{i} S_{ii} + 3 \lambda \sum_i \delta S_{ii},
\sum_{i} T_{ii}= \sum_{i} (\mu + 3 \lambda) S_{ii},

revert to the summation convention
T_{ii}= (\mu + 3 \lambda) S_{ii}.
 
Thanks a lot I see now - I thought the rules were that if i rename Skk to Sii, I must also change Tii to Tkk.
 
Question: A clock's minute hand has length 4 and its hour hand has length 3. What is the distance between the tips at the moment when it is increasing most rapidly?(Putnam Exam Question) Answer: Making assumption that both the hands moves at constant angular velocities, the answer is ## \sqrt{7} .## But don't you think this assumption is somewhat doubtful and wrong?

Similar threads

  • · Replies 5 ·
Replies
5
Views
1K
  • · Replies 10 ·
Replies
10
Views
3K
  • · Replies 4 ·
Replies
4
Views
2K
  • · Replies 9 ·
Replies
9
Views
2K
  • · Replies 1 ·
Replies
1
Views
2K
  • · Replies 12 ·
Replies
12
Views
2K
  • · Replies 2 ·
Replies
2
Views
3K
  • · Replies 2 ·
Replies
2
Views
1K
  • · Replies 7 ·
Replies
7
Views
659
  • · Replies 7 ·
Replies
7
Views
2K